第55届(2014)IMO中国国家队选拔考试一(第一天)试题及其解答
标签:
流年 |
分类: 奥赛 |

1.(ThirdTimeLucky )Lemma 1: Suppose http://data.artofproblemsolving.com/images/latex/0/0/c/00c1bff9de8d4fc009abbfd696151a6d58535852.gif meets http://data.artofproblemsolving.com/images/latex/0/f/4/0f4d56d1e20778bf2e1052ecb3219509238fb660.gif at http://data.artofproblemsolving.com/images/latex/a/7/e/a7ee38bb7be4fc44198cb2685d9601dcf2b9f569.gif . Then http://data.artofproblemsolving.com/images/latex/9/d/4/9d4fc26a50bdeb6214f335053db2509f2e6393c1.gif are collinear.
Proof: It suffices to show http://data.artofproblemsolving.com/images/latex/8/3/5/835aa494b205bb7a512a2f23357cb52ef2fe9f3c.gif is cyclic and http://data.artofproblemsolving.com/images/latex/b/8/6/b86bd135c70e92d6b2051a988af8d8fd3b21cb67.gif isosceles and thus making http://data.artofproblemsolving.com/images/latex/b/7/8/b7801d3332d72be00bb742186513047b1a0d3ac0.gif the perpendicular bisector of http://data.artofproblemsolving.com/images/latex/5/b/3/5b3e3191fd7659c5e63ab6620a903048b48bcb56.gif . Let http://data.artofproblemsolving.com/images/latex/d/5/3/d53599f21923e6269c18fd2b78a97c088222d14a.gif . Then http://data.artofproblemsolving.com/images/latex/5/5/3/5536fd12b5f28e7a1f81673b47dd919f3a5f27d6.gif . If http://data.artofproblemsolving.com/images/latex/0/3/f/03f7d5a32f20b3e09718a39837b468e1075fdff8.gif hits http://data.artofproblemsolving.com/images/latex/6/d/9/6d95c1847219c633950f8f1ceca9761315abfc19.gif at http://data.artofproblemsolving.com/images/latex/6/8/6/686cf6635679037d0da6c7180e2943ac3b9e23e6.gif , we immediately get http://data.artofproblemsolving.com/images/latex/9/8/b/98bc72d077f73e3a1df9623a638ebb300954fbcb.gif is cyclic and http://data.artofproblemsolving.com/images/latex/9/d/4/9d4fc26a50bdeb6214f335053db2509f2e6393c1.gif are collinear,
Similarly, if http://data.artofproblemsolving.com/images/latex/b/1/f/b1fb3bec6fdb22e19a94fe4c6c4481ccba2ee9f0.gif at http://data.artofproblemsolving.com/images/latex/d/1/6/d160e0986aca4714714a16f29ec605af90be704d.gif , then http://data.artofproblemsolving.com/images/latex/0/e/6/0e652b24136b04a4e6b0c9c54f027a0e1ba8c00b.gif are collinear.
Now, http://data.artofproblemsolving.com/images/latex/3/c/7/3c79a7fae6a8e00fd9a4a606df1498166bf94880.gif is cyclic, which gives http://data.artofproblemsolving.com/images/latex/1/4/4/144e2def31a00854c9a12dc7afd3b54d23d7121b.gif . Since http://data.artofproblemsolving.com/images/latex/5/4/8/5489d6b40014c4bd9c71fd09bb9a95945b63d103.gif is also cyclic, http://data.artofproblemsolving.com/images/latex/8/5/2/8524da9c90c7457f90cb374edee1542404724d3e.gif. But ofcourse http://data.artofproblemsolving.com/images/latex/e/1/4/e1455ae6ae0f45d4cf0a96e69a74334a1c6c85ae.gif.
1.(XmL )My
proof doesn't require proving the collinearity that ThirdTimeLucky
mentioned(even though it's pretty apparent when drawn). It's clear
that http://data.artofproblemsolving.com/images/latex/d/b/6/db6cb0096bc200144861ee6073f8e207f9574b1c.gif are concyclic, since
http://data.artofproblemsolving.com/images/latex/a/e/3/ae376f9cbd130e6a792483a938f373223d8ecc2c.gif. This means we only
have to prove http://data.artofproblemsolving.com/images/latex/c/3/1/c3156e00d3c2588c639e0d3cf6821258b05761c7.gif is concyclic with
http://data.artofproblemsolving.com/images/latex/2/6/1/2611def36987afec6083f8f9bc62a9e3c85ab010.gif
. Through http://data.artofproblemsolving.com/images/latex/e/0/1/e0184adedf913b076626646d3f52c3b49c39ad6d.gif
we construct a line parallel to http://data.artofproblemsolving.com/images/latex/0/f/4/0f4d56d1e20778bf2e1052ecb3219509238fb660.gif
and it intersects http://data.artofproblemsolving.com/images/latex/6/d/9/6d95c1847219c633950f8f1ceca9761315abfc19.gif
at http://data.artofproblemsolving.com/images/latex/c/0/3/c032adc1ff629c9b66f22749ad667e6beadf144b.gif
, since http://data.artofproblemsolving.com/images/latex/b/0/6/b06c7e0cde491bafedf4ecaa7ae7f85bf550adfa.gifis a rectangle. Since
http://data.artofproblemsolving.com/images/latex/c/3/1/c3156e00d3c2588c639e0d3cf6821258b05761c7.gif
is concyclic with http://data.artofproblemsolving.com/images/latex/2/6/1/2611def36987afec6083f8f9bc62a9e3c85ab010.gif
and we are done.
2.(dgrozev)Let http://data.artofproblemsolving.com/images/latex/d/e/0/de07ec02d309c73f8b02483d5e0eb4b3a0d834cb.gif.
Obviously http://data.artofproblemsolving.com/images/latex/c/c/8/cc84db5dc0e7b9169d2c34d1f3cbc12cd63e565c.gif.
Suppose Suppose:
http://data.artofproblemsolving.com/images/latex/7/9/7/797abc23557866c5e1e47a520c62f0d876f73926.gif
http://data.artofproblemsolving.com/images/latex/8/5/5/8555093873a4bd74395c87ade08d9eee8f9b7087.gif
http://data.artofproblemsolving.com/images/latex/3/1/0/3100265ed9f7a342ef296b6455a3804df5ec4929.gif
It can be checked that:
(i)
(ii)
(iii)
(iiii)
Therefore
3.(61plus)Lemma: For
a prime http://data.artofproblemsolving.com/images/latex/5/1/6/516b9783fca517eecbd1d064da2d165310b19759.gif,
if http://data.artofproblemsolving.com/images/latex/a/4/4/a44ff445ea60f30bcdacdd97084c66ece8f26d77.gif,
then http://data.artofproblemsolving.com/images/latex/7/6/8/768f8352414999c25c518730ff516616e6ee5194.gif.
Proof: Note that when http://data.artofproblemsolving.com/images/latex/4/1/9/419c4295a265c6cba14e4237d697ef6a1948ee28.gif,
then http://data.artofproblemsolving.com/images/latex/f/3/6/f36c1bc88cc6de0b322bb59979985aed600f3c9d.gif
Construct the following infinite sequence of numbers:http://data.artofproblemsolving.com/images/latex/3/4/6/34663d14eefe0496d8967d0cd14b500a09fed220.gif.
Note that http://data.artofproblemsolving.com/images/latex/3/d/5/3d5553eaf7e7b3bcb1a92ebb623de8951fdd32ce.gif for
all http://data.artofproblemsolving.com/images/latex/6/f/e/6fe631d550f98de6c2527d4d858f038604fa234d.gif,
and http://data.artofproblemsolving.com/images/latex/e/3/c/e3cbe635e202b5e06d6f155608ae24cf4e2271da.gif for
all http://data.artofproblemsolving.com/images/latex/3/c/e/3ced74cab564cd52f7d97cced08fe9700afff503.gif for
all http://data.artofproblemsolving.com/images/latex/e/e/b/eebecf421c4d33eeab4a0c4da6c20ed8d49e6c6c.gif.
Hence there are infinitely many http://data.artofproblemsolving.com/images/latex/2/f/1/2f17c46d99321acaa8de5fcfad1c853e8b55609c.gif.
Now for any http://data.artofproblemsolving.com/images/latex/1/1/f/11f6ad8ec52a2984abaafd7c3b516503785c2072.gif,
suppose there exist prime http://data.artofproblemsolving.com/images/latex/5/1/6/516b9783fca517eecbd1d064da2d165310b19759.gif such
that http://data.artofproblemsolving.com/images/latex/9/a/7/9a75dcf2deb37acd35c82e413bcc496d62f7f613.gif.
We can find http://data.artofproblemsolving.com/images/latex/d/0/c/d0ccf51cfe6728032bed8393af726ff5bbfe6537.gif, such
that http://data.artofproblemsolving.com/images/latex/6/c/6/6c63e602a0ef08ecaba25812c1f34c9336ed3896.gif,
and http://data.artofproblemsolving.com/images/latex/b/5/8/b584458444c4d405a40678abb3e67e208d883052.gif.
Then by Chinese Remainder Theorem, there
exist http://data.artofproblemsolving.com/images/latex/4/d/c/4dc7c9ec434ed06502767136789763ec11d2c4b7.gif such
that http://data.artofproblemsolving.com/images/latex/8/d/f/8df9ba157d146318863524a022d95506dacc5d2c.gif We
conclude that http://data.artofproblemsolving.com/images/latex/3/1/6/316936d278b6679d119b35528520e1fb95eaaa64.gif.
But http://data.artofproblemsolving.com/images/latex/c/5/c/c5cdc3d3e86752cf831766494880e2c30abca120.gif,
contradiction.
Construct the following infinite sequence of numbers:
Remainder of solution
Note that for http://data.artofproblemsolving.com/images/latex/b/2/0/b208abfdaf0a40f6093c78bf37ac6300ece1d718.gif,
suppose there exist http://data.artofproblemsolving.com/images/latex/3/4/4/34498b9401a5a771f51ef614ed7be2b46e50e692.gif.
Then http://data.artofproblemsolving.com/images/latex/e/0/f/e0f54ca924dcdbd2fd06c1ab379384a506185fc7.gif,
so http://data.artofproblemsolving.com/images/latex/e/f/f/eff683f88d83e21ea85c7937301f960d884207da.gif,
contradiction, hence http://data.artofproblemsolving.com/images/latex/3/e/0/3e03f4706048fbc6c5a252a85d066adf107fcc1f.gif is
injective for http://data.artofproblemsolving.com/images/latex/7/3/7/737b45b9a8f2b2fea357675c90c769ca2c2f6775.gif.
Hence if we show that there existhttp://data.artofproblemsolving.com/images/latex/a/6/4/a6464f6f31a378bca7899feaf2d699209979434f.gif, such
that http://data.artofproblemsolving.com/images/latex/6/3/2/632d96b7f3ab0d65a6f61b523e0a7d0db386bc07.gif for
all http://data.artofproblemsolving.com/images/latex/d/0/c/d0ccf51cfe6728032bed8393af726ff5bbfe6537.gif,
then http://data.artofproblemsolving.com/images/latex/f/1/d/f1dbdd3a5b38fafd9f78c329171bfe72dea6faf8.gif for
all http://data.artofproblemsolving.com/images/latex/7/a/a/7aa0d5db36fd7b9800ad3c90cecb6d9afcc29e45.gif.
It remains to show that there are infinitely many
such http://data.artofproblemsolving.com/images/latex/3/c/3/3c363836cf4e16666669a25da280a1865c2d2874.gif.
Consider http://data.artofproblemsolving.com/images/latex/3/c/3/3c363836cf4e16666669a25da280a1865c2d2874.gif such
that http://data.artofproblemsolving.com/images/latex/5/7/0/57005dd7245bdc1e4ddb5941a70fdd09e5605597.gif for
all http://data.artofproblemsolving.com/images/latex/4/0/6/406158a3dfa33eda1262bbe11614ca4a289862eb.gif are
large enough distinct primes. http://data.artofproblemsolving.com/images/latex/3/c/3/3c363836cf4e16666669a25da280a1865c2d2874.gif exists
from Chinese Remainder Theorem. Then http://data.artofproblemsolving.com/images/latex/f/4/a/f4a5baac6440b9492ad09775dcabc1101fcd0505.gif for
all http://data.artofproblemsolving.com/images/latex/d/0/c/d0ccf51cfe6728032bed8393af726ff5bbfe6537.gif,
unless http://data.artofproblemsolving.com/images/latex/8/e/4/8e4587fc82ce6377530643c5622b41e53cdf3dd3.gif.
Hence from lemma we get http://data.artofproblemsolving.com/images/latex/6/3/2/632d96b7f3ab0d65a6f61b523e0a7d0db386bc07.gif for
all http://data.artofproblemsolving.com/images/latex/d/0/c/d0ccf51cfe6728032bed8393af726ff5bbfe6537.gif. As we can always pick
larger primes, we can have infinitely many
such http://data.artofproblemsolving.com/images/latex/3/c/3/3c363836cf4e16666669a25da280a1865c2d2874.gif.
Hence if we show that there exist
注:(USA TSTST 2012, Problem
3)Let http://data.artofproblemsolving.com/images/latex/5/3/6/536c886d7863df5a4e250a73547be5d968c290c7.gif be
the set of positive integers. Let http://data.artofproblemsolving.com/images/latex/b/f/e/bfefc53fb39bf125cfb72d6037e1773cbbfe02c2.gif be
a function satisfying the following two
conditions:
(a) (b)
Prove that for any natural number
前一篇:电影流痕(72)

加载中…